Sunteți pe pagina 1din 180

Screening & Prevention

All of the following patients are more likely to be susceptible to tetanus except: A. A 70-year-old man with diabetes. B. A 40-year-old ex-Marine sergeant. C. A 65-year-old lady who works in her garden. D. A 23-year-old Mexican migrant worker.

Physicians giving multiple vaccinations to a 12- month-old infant should do all of the following except: A. Ask about contraindications or adverse reactions to previous vaccines. B. Give each vaccine in a separate injection at a separate site. C. Give MMR 2 weeks after the varicella vaccine so the child does not receive as many shots. D. Give the current Vaccine Information Statement for each vaccine administered.

Inactivated influenza vaccine is indicated for all of the following patients except: A. A 60-year-old lady with congestive heart failure. B. A healthy 52-year-old chicken farmer who is allergic to eggs. C. A 45-year-old lady with Type 2 diabetes mellitus. D. A 10-year-old child with asthma.

Which of the following statements is true regarding the following vaccine preventable diseases in the United States? A. Almost one half of an increasing number of pertussis cases occur in adolescents and adults. B. The number of cases of invasive disease due to Haemophilus influenzae Type b is increasing. C. Because chickenpox is always a benign disease, children should be exposed to other children with chickenpox so they will develop disease and become immune. D. Infections due to invasive pneumococcal disease in children less than 2 years of age has not decreased.

Which of the following are indicative of moderate risk for colorectal cancer? A. History of colorectal cancer or adenomas. B. History of endometrial, ovarian or breast cancer. C. A first-degree relative with colorectal cancer or a history of polyps before age 60. D. All of the above.

Which one of the following screening methods does not require bowel preparation and sedation? A. Sigmoidoscopy. B. Double-contrast barium enema and virtual colonoscopy. C. Colonoscopy. D. FOBT and stool-based DNA.

Which screening options must be followed with an examination of the entire colon if initial results are positive? A. Annual fecal occult blood test (FOBT). B. Annual FOBT with flexible sigmoidoscopy. C. Double-contrast barium enema. D. All of the above.

Which of the following interventions has/have been shown to be useful in helping patients stop smoking? (choose all that apply) A. Physician advice to stop smoking. B. Proactive telephone counseling. C. Assistance in the development of stress management and problemsolving skills. D. Regular follow-up on the patient s progress.

Which of the following statements about regular physical activity is/are true? (choose all that apply) A. Physical activity programs should be individualized for each patient. B. Patients who have a sedentary lifestyle should begin with a vigorous exercise program. C. Regular physical activity lowers a person s blood glucose levels and improves his or her insulin sensitivity. D. Adequate hydration is important during and after regular physical activity.

Which of the following statements regarding lifestyle choices is/are true? (choose all that apply) A. People who smoke have twice the MI risk of nonsmokers. B. Approximately 40 percent of people in the United States eat the recommended minimum of five servings of fruits and vegetables each day. C. People who are not physically active are twice as likely to develop heart disease. D. In 2000, more than 50 percent of people in the United States were obese (BMI = 30.0 kg per m2).

The first in the series of haemophilus influenzae type B conjugate vaccine should be routinely administered to children when they reach the age of A. B. C. D. E. 2 months 4 months 6 months 12 - 15 months 18 months

Which one of the following causes the most injuries requiring medical treatment for women? A. B. C. D. E. Automobile accidents Muggings Self-inflicted injury Domestic violence Rape

Which one of the following risk factors qualifies a patient as having a CHD risk equivalent (20% ten-year risk of CHD) according to the NCEP ATP III guidelines? A. Cigarette smoking. B. Diabetes mellitus. C. Obesity. D. Family history of premature CHD.

Which one of the following is the best predictor of reaching long-term treatment goals? A. Patient age. B. Socioeconomic level. C. Once-daily medication dosing. D. Planned follow-up visits.

Current polio vaccination recommendations call for vaccination with: A. An all-oral polio vaccine (OPV) regimen. B. An all-inactivated poliovirus vaccine (IPV) regimen. C. Two doses of IPV followed by 2 doses of OPV. D. Two doses of OPV followed by 2 doses of IPV. E. None of the above. Because of the success of the polio vaccine, the risk of developing vaccine-associated infection from the oral polio vaccine (OPV) is higher than the risk of contracting the disease from wild-type virus. Because of this risk, the ACIP changed its recommendations for polio immunization. The new recommendation is to use only inactivated poliovirus (IPV), and the OPV should be eliminated in most cases.

Incorporating preventive services into patients visits for acute and chronic illnesses increases the length of an average office visit by: A. 2 minutes. B. 4 minutes. C. 6 minutes. D. 8 minutes. E. 10 minutes. Physicians and their office personnel are frequently so busy treating patients acute and chronic illnesses that they perceive that there is insufficient time to discuss prevention. However, researchers report that preventive services increase office visits by an average of only 2 minutes.

To prevent sudden infant death syndrome, parents of a newborn infant should be told that the recommended sleep position for infants is: A. Supine. B. Prone. C. On their sides. D. On a soft surface. E. Well supported by pillows. Recently, the issue of sleep position and SIDS has changed what physicians recommend to parents. Studies found that infants sleeping in the supine position were less likely to develop SIDS. Parents should be instructed to place their infant in the supine position for sleep. In addition, parents should be advised to avoid using soft surfaces, pillows, or quilts in the infant s bed.

Sexually active adolescent women should be screened for: A. Chlamydia. B. Bacterial vaginosis. C. Trichomonas vaginalis. D. All of the above. E. None of the above. In 1998, almost 88% of all high school girls had experienced intercourse. Most medical groups also recommend screening for chlamydia, gonorrhea, and syphilis at the time of each Papanicolaou s smear.

You are conducting pre-participation physicals for a high school varsity football team. You decide to use the recommendations of the U.S. Preventive Services Task Force for testicular screening. According to these guidelines, these young men should have a routine testicular examination: A. Yearly starting at age 15 years. B. Every other year starting at age 17 years. C. Every 3 years post puberty. D. Every 5 years starting at age 15 years. E. Never; no routine testicular screening is recommended.
Testicular examinations are controversial. The ACS recommends an examination every 3 years for men beginning at age 18 years while the American Urological Association recommends yearly examinations starting at age 15 years. The U.S. Preventive Services Task Force (USPSTF) advises against screening because a large number of false-positive results are likely, and, therefore, examinations are not in the best interest of individuals who are at low risk in this age group. People with risk factors for testicular cancer, such as cryptorchidism, orchiopexy, or testicular atrophy, may benefit from periodic screening.

All of the following are risk factors for cervical cancer except: A. Smoking. B. Intercourse with a high-risk male partner. C. Age of first intercourse after age 21 years. D. Multiple sexual partners. E. Infection with human papillomavirus.

Risk Factors for Cervical Cancer ! Early age of rst intercourse (aged 20 years or younger)! Multiple sexual partners! History of or current human papillomavirus infection! Smoking! Lower socioeconomic class! Intercourse with a man who has had multiple sexual partners

A 74-year-old patient should routinely be screened for colorectal cancer with an annual: A. Digital rectal examination. B. Air-contrast barium enema. C. Flexible sigmoidoscopy. D. Colonoscopy. E. Fecal occult blood test.

Screening for colorectal cancer is recommended using annual fecal occult blood testing, sigmoidoscopy every 5 years, or colonoscopy every 10 years.

Which of the following individuals is likely to be more successful in committing suicide? A. An older white man. B. An older white woman. C. An older African American man. D. An older African American woman. E. An older Asian man.

An assessment for suicide risk is important in depressed elderly patients. Elderly white men are more often successful in committing suicide than other demographic groups.

As medical director of a large corporation, you are asked to develop recommendations for periodic employee health examinations. About 20% of these employees (males, age 40 to 60, smoking one or more packs of cigarettes daily) can be considered at increased risk for lung cancer. Which one of the following strategies for the early detection of lung cancer in asymptomatic individuals would you recommend? A. No strategy has been shown to be effective B. Chest roentgenogram and sputum cytology every 4 months for high-risk employees C. Annual chest roentgenogram and sputum cytology every 4 months for high-risk employees D. Annual chest roentgenogram and sputum cytology for all employees E. Annual chest roentgenogram for all employees Screening for lung cancer is not recommended by the American Cancer Society, the American College of Radiology, the National Cancer Institute, the U.S. Preventive Services Task Force, or the Canadian Task Force on the Periodic Health Examination. None of the published large-scale studies of the effects of screening for lung cancer has so far demonstrated that screening reduces lung cancer mortality, even in high-risk persons.

Influenza has been reported in your community for the last 2 weeks. A nursing home where you have many patients calls to report that influenza has struck several residents. Your patients received influenza vaccine several weeks ago. Which one of the following statements regarding this situation is true? A. You should give a second dose of influenza vaccine to all patients B. Amantadine causes frequent central nervous system toxicity in the elderly and should not be given to immunized elderly patients who develop fever C. The influenza vaccine is highly efficacious in adults and prevents influenza in over 95% of patients when the infecting strain is included in the vaccine D. Influenza vaccination reduces morbidity and mortality in the elderly, but prevents

influenza infection in less than 70% of vaccinated elderly patients

Influenza vaccine has been shown to prevent influenza in well under 70% of nursinghome patients, with some studies showing only 30% to 40% efficacy. Nevertheless, these residents benefit greatly from a reduced morbidity and mortality, due to less severe infection and fewer complications. Many experts recommend amantadine prophylaxis for all residents when there is an influenza outbreak in the nursing home, regardless of vaccination status. There is an increased incidence of central nervous system toxicity in the elderly, but this should not preclude use of the drug to treat or prevent influenza. There is no indication for revaccinating patients.

Which one of the following is the greatest risk factor for developing breast cancer? A. Menopause before the age of 45 years B. Bilateral premenopausal breast cancer in a first-degree maternal relative C. Hyperplastic lesions present on breast biopsy (i.e., atypical lobular hyperplasia) D. Multiparity Positive risk factors for breast cancer include nulliparity, a history of breast cancer in a first-degree maternal relative, and late menopause (>55 years). Multiparity, atypical lobular hyperplasia, and early menopause are not significant risk factors.

Which one of the following statements is true concerning recommendations about health care for adolescents?
A. For confidentiality reasons, parents should be excluded from the adolescent health care delivery process B. Routine screening of all adolescents for tuberculosis with a yearly skin test is recommended C. Immunization of unimmunized adolescents against hepatitis B is not recommended unless they are at high risk D. Sexually active female adolescents should have a Papanicolaou (Pap) test at 3-year intervals E. All sexually active adolescents should be screened for sexually transmitted diseases According to the Guidelines for Adolescent Preventive Services (GAPS), sexually active adolescents should be screened for sexually transmitted diseases, and sexually active female adolescents should have a Papanicolaou test every year. GAPS also suggests following the recommendations of the Advisory Committee on Immunization Practices (ACIP) for universal immunization of adolescents against hepatitis B. Only high-risk adolescents need screening for tuberculosis. Confidential care for adolescents is important, and policies need to be developed concerning the way parents are involved in that care.

Contraindications to immunization include: A. B. C. D. E. Antibiotic use Anaphylactic reaction to vaccine High fever with prior administration All of the above B and C only

Risk factors for prostate cancer include: A. B. C. D. E. Family history of breast cancer Black ancestry Dietary fat intake All of the above B and C only

Sprains, Strains, & Sports Med

In adults under 30, the most common organism causing acute infectious arthritis is A. B. C. D. Neisseria gonorrhoeae Streptococcus viridans Streptococcus pneumoniae Staphylococcus aureus

Which one of the following conditions is the most frequent cause of sudden cardiac death in athletes under age 35? A. B. C. D. E. Myocarditis Hypertrophic cardiomyopathy Ruptured aorta Atherosclerotic coronary artery disease Coronary anomalies

In athletes under age 35, hypertrophic cardiomyopathy accounts for 36% of sudden cardiac deaths. Coronary anomalies, especially the anomalous origin of the left main coronary artery from the right sinus of Valsalva, is next most frequent at 19%. Ruptured aorta and myocarditis are considerably less frequent at 5% and 3% respectively. The least common of those listed is atherosclerotic coronary artery disease, which causes only 2% of sudden cardiac deaths in this group. After the age of 35, however, this disorder is the most common etiology by a wide margin.

When examining a patient with classic rheumatoid arthritis, which one of the following physical findings is most consistent with the diagnosis? A. B. C. D. E. Janeway lesions and Osler's nodes Bouchard's and Heberden's nodes Synovitis of multiple metacarpophalangeal joints Telangiectasias of the periungual skin Linear pitting of the fingernails

Classic rheumatoid arthritis patients exhibit the following physical findings:

swelling and redness; synovitis of multiple small hand, wrist, and foot joints; rheumatoid nodules on extensor surfaces; ulnar deviation and subluxation of the proximal phalanges; cocking-up of the toes; fusiform swelling of the fingers; swan neck and boutonniere deformities of the fingers; and synovial hypertrophy of large joints. Pitting of the nails is seen in psoriasis. Periungual telangiectasias are seen in CREST syndrome and progressive systemic sclerosis. Bouchard's and Heberden's nodes are seen in patients with degenerative joint disease. Janeway lesions and Osler's nodes are signs of acute bacterial endocarditis.

A 22- year old man comes to the office with pain and swelling in his right knee. He was playing soccer, but he says he was not hit in the knee. He does remember running, turning sharply, and then stopping quickly while attempting to kick a goal just before he felt the pain. Within an hour, his knee was swollen. On physical exam, you find that he has a large effusion. What type of knee injury would you now suspect? A. Torn medial meniscus B. Ruptured posterior cruciate ligament C. Ruptured anterior cruciate ligament D. Torn medial collateral ligament E. Torn lateral collateral ligament
Quick stops and sharp cuts or turns often create significant deceleration forces that can rupture the anterior cruciate ligament in the knee. It can also occur with hyperextension of the knee. Rapid onset, usually within 2 hours of the injury, of a large, tense effusion suggests rupture of the anterior cruciate ligament or fracture of the tibial plateau. Slower onset of 24-36 hours, of a mild or moderate effusion can be seen with meniscal injury or ligamentous sprain.

A father brings in his 15-year old son who is complaining of a painful right wrist after he fell on it while playing softball. He has pain over the anatomic snuffbox. You suspect a fracture of the scaphoid bone, but radiographs of the right wrist do not reveal one. Which one of the following options would be most appropriate at this time? A. Explain that radiographs are negative for fracture, treat the boy for pain, and ask them to return if the pain persists or worsens. B. Put a short arm-thumb spica cast on the boy and re-evaluate in 2 weeks C. Refer him to a physical therapist to relieve the pain and maintain range of motion. D. Obtain an ultrasound exam with routine equipment to detect an occult fracture.
Anatomic snuffbox tenderness is 90% sensitive but only 40% specific for a scaphoid fracture. Initial radiographs do not always show a fracture, especially those that are nondisplaced. As nonunion is a complication of a scaphoid fracture due to poor blood supply and unstable fracture fragments, it would be reasonable to place a short armthumb spica cast and have him return in 2 weeks for re-evaluation and follow-up X-rays. Ultrasound is not recommended in the initial evaluation of a suspected scaphoid fracture. Other imaging such as MRI and bone scintigraphy have proven accurate for detecting occult scaphoid fracture.

A 15-year-old boy is brought to the emergency department 1 hour after an episode of syncope while running in a 400-meter race. He had a similar episode 2 years ago. His mother and maternal first cousin died suddenly at the ages of 32 and 17 years, respectively. Examination shows abrasions of the face, hands, and knees. Cardiac and neurologic examinations show no abnormalities. Which of the following is the most appropriate next step in diagnosis? (A) Tilt test (B) CT scan of the head (C) ECG (D) EEG (E) Lumbar puncture

A 43-year-old woman who works as a seamstress has a 3-month history of pain, weakness, and numbness of the right hand that is worse at night. Examination shows hypoesthesia and atrophy of the thenar eminence. These findings are most likely caused by compression of which of the following nerves? (A) Axillary (B) Brachial cutaneous (C) Median (D) Radial (E) Ulnar

A 12-year-old girl is referred to the clinic by the school nurse for evaluation of scoliosis. The girl's scoliosis was detected during a routine screening examination at the school, and it appears to be mild (curve less than 10 degrees). She is athletic and is otherwise in good health. During the physical examination, particular attention should be given to which of the following? (A) Arm length (B) Blood pressure (C) Body weight (D) Cardiac examination (E) Stage of pubertal development

A 22-year-old man returns to the office for a follow-up visit 4 weeks after being diagnosed with mononucleosis. He is a college football player and the football season is just beginning. At his initial visit, he had a sore throat, nausea and a temperature of 39.4EC (103.0EF). Physical examination at that time showed pharyngitis, submandibular and posterior cervical lymphadenitis, and splenomegaly. Complete blood count (CBC) done at that time showed a leukocyte count of 18,000/mm3 with an elevated number of monocytes, and a Monospot test was positive. The course of his illness has been uncomplicated, but today he still complains of some residual fatigue. Before you examine him, he asks you, "When can I play again?" Which of the following is the most appropriate response to the patient regarding when he may return to contact sports? (A) Next season (B) When a Monospot test is negative (C) When he is no longer symptomatic (D) When his CBC is normal (E) When his physical examination is normal

Menstrual Problems/STDs

A 57-year-old white female is 4 years post menopause and is beginning to experience dyspareunia. A pelvic examination is normal for her age. She is very concerned about maintaining
her sexual ability. You would suggest A. B. C. D. E. estrogen therapy and maintaining regular sexual activity a vinegar douche sitz baths triple sulfa vaginal cream sexual abstinence for 3 months to allow vaginal healing

The most common clinical symptom affecting sexual activity in postmenopausal women is dyspareunia secondary to a thin and friable vaginal mucosa, mainly from reduced estrogen effect and decreased lubrication. Regular sexual activity retards the effects of aging on sexual organs and responses. Estrogen replacement therapy is also helpful.

A 35-year-old nulliparous white female has taken oral contraceptives for 5 years. Her physical examination is unremarkable. Continued use of oral contraceptives is likely to lower her risk for A. B. C. D. E. malignant melanoma lung cancer leukemia ovarian cancer renal cell carcinoma

Recent studies strongly support the effect of oral contraceptives in preventing ovarian cancer. In fact, some authorities believe that their prophylactic use should be considered in women at high risk for the disease. Nulliparity and a positive family history are considered to be risk factors for ovarian cancer. The longer these women are on the pill, the greater their risk reduction; in addition, the protective effect may persist for as long as 10 years after oral contraceptives are discontinued. The risk of the other neoplasms mentioned is not lowered as dramatically by oral contraceptive use.

Long-term use of oral contraceptives containing a low-dose estrogen/progestin combination has been shown to increase the risk of which one of the following conditions? A. B. C. D. E. Hyperlipidemia Hypertension Endometrial cancer Ovarian cancer Breast cancer

Low-dose combination oral contraceptives may increase the risk of hypertension by up to 5% over a 5-year period. They have not been shown to increase the risk of breast cancer, and may decrease the risk of ovarian and uterine cancer. The low-dose forms show minimal effects on serum lipids, but may cause a small increase in HDL and VLDL levels.

Which one of the following has been shown to improve symptoms of premenstrual dysphoria? A. B. C. D. E. Diazepam (Valium) Thioridazine (Mellaril) Medroxyprogesterone acetate (Provera) Fluoxetine (Prozac) Ibuprofen (Motrin)

Of the medications listed, only fluoxetine has been shown in a randomized trial to ameliorate symptoms of premenstrual dysphoria in premenstrual tension syndrome.

A 16-year-old, sexually active, nonpregnant female has proven chlamydial urethritis and mucopurulent cervicitis. Which one of the following is the recommended treatment?
A. B. C. D. E. Ceftriaxone (Rocephin), 250 mg intramuscularly in a single dose Azithromycin (Zithromax), 1 g orally Ciprofloxacin (Cipro), 500 mg twice a day for 3 days Metronidazole (Flagyl), 2 g orally in a single dose Double-strength trimethoprim/sulfamethoxazole (Bactrim DS, Septra DS)

The results of clinical trials indicate that azithromycin and doxycycline are equally efficacious for chlamydial infection. Azithromycin may be useful when compliance is uncertain. Doxycycline is less expensive, but azithromycin may be more cost effective because it provides single-dose, directly observed therapy. Ceftriaxone is indicated for gonococcal infection. Metronidazole is indicated for recurrent urethritis or bacterial vaginosis. Ciprofloxacin is recommended for chancroid.

A 25-year-old male visits your office concerned about a painless ulcer on the glans of his penis. After appropriate examination and testing you diagnose primary syphilis and treat him with 2.4 million units of benzathine penicillin intramuscularly in a single dose. Eight hours later, while you are working the evening clinic, he returns because he has a fever of 100.6 F and a bad headache, which he rarely gets. He says he "aches all over." Which one of the following is the most appropriate action at this time? A. Reassure and prescribe antipyretics B. Add doxycycline (Vibramycin), 100 mg orally twice a day for 14 days C. Perform a spinal tap D. Order a CT scan of his head E. Obtain three blood cultures from different sites at 30-minute intervals
This man is experiencing the Jarisch-Herxheimer reaction-an acute, transient febrile reaction that occurs within the first few hours after treatment for syphilis. The illness peaks at 6-8 hours, and disappears within 12-24 hours after therapy. Temperature elevation is usually low grade, and there is often associated myalgia, headache, and malaise. It is usually of no clinical significance and may be treated with salicylates in most cases. The pathogenesis of the reaction is unclear, but it may be due to liberation of antigens from the spirochetes.

Dysfunctional uterine bleeding is most commonly caused by: A. B. C. D. E. Neoplasms of the genital tract The failure of cyclical ovulation with a normal genital tract Thyroid disorders Oral contraceptive use Endometrial polyps

By definition, dysfunctional uterine bleeding (DUB) is bleeding in the absence of organic disease and is therefore a diagnosis of exclusion. The most common cause of DUB is the failure of the normal, cyclical ovulation process to occur, which results in an unstable endometrial lining. This unstable lining can shed at irregular intervals producing Non cyclical uterine bleeding.

A 22-year old woman comes to your office complaining of vaginal discharge. After taking her medical history, performing a physical examination, and doing a vaginal wet mount, you make the diagnosis of trichomoniasis. Which one of the following pharmacologic regimens is the most appropriate treatment for this women s condition? A. B. C. D. E. Metronidazole, 2 gm po as a single dose Metronidazole, 0.75% intravaginal gel BID Clindamycin, 2% intravaginal cream once daily Clindamycin, 300 mg po TID Fluconazole 150 mg po as a single dose

Oral metronidazole is the current treatment for trichomoniasis. It can be given as a 2-gm single dose, as a 250 mg dose TID for 7 days, or as a 500 mg dose BID for 7 days. Intravaginal therapy is not effective. Intravaginal metronidazole or clindamycin (which can also be taken orally) are therapies for bacterial vaginosis. Single-dose fluconazole is a treatment for candidiasis.

Of the following pharmacologic interventions, all except _______ are successful treatments for dysmenorrhea. A. B. C. D. E. Danazol (Danocrine) acetaminophen Medroxyprogesterone contraceptive injection (Depo-Provera) Levonorgestrel-releasing intrauterine system (Mirena) Oral contraceptive pill

Risk factors for dysmenorrhea include young age, depression, heavy menses, nulliparity, and smoking. NSAIDS are the best initial therapy, having a direct analgesic effect and decreasing menstrual flow by inhibiting prostaglandin syntheses. The next best studied and beneficial therapies are effective by virtue of the fact that they inhibit the menstrual cycle. These include danazol, leuprolide acetate, medroxyprogesterone, and the extended cycle oral contraceptives. Less studied modalities include COX-2 inhibitors, levonorgestel-releasing intrauterine system, thiamine and Vit E supplementations, acupuncture or acupressure, and fish oil supplements. The evidence concerning the efficacy of acetaminophen in dysmenorrhea is limited and not conclusive with respect to placebo.

All of the following except ________ are diagnostic criteria for polycystic ovary syndrome. A. Clinical or biochemical evidence of hyperandrogenism. B. Oligo-ovulation C. Exclusion of disorders such as congenital adrenal hyperplasia or hyperprolactinemia D. Polycystic ovaries on ultrasound

Ovaries with multiple small follicular cysts are found in 16-25% of healthy women and in those who have amenorrhea from causes other than polycystic ovary syndrome (PCOS). About 80% of women with hyperandrogenism have polycystic ovaries. Other disorders such as adrenal hyperplasia or hyperprolactinemia must be excluded. Oligo-ovulation can also be an indication. A finding of polycystic ovaries on ultrasound is not essential for making the diagnosis of PCOS.

A sexually active 14-year-old girl is brought to the emergency department because of the acute onset of right lower quadrant abdominal pain and loss of appetite for 18 hours. She has had no nausea or vomiting. Her temperature is 38.9EC (102EF). Bimanual pelvic examination shows cervical exudate and tenderness on cervical motion. There is bilateral lower quadrant tenderness. Her leukocyte count is 21,300/mm3. Her serum -Hcg concentration is within normal limits. Which of the following is the most appropriate next step in management? (A) Antibiotic therapy (B) Meckel scan (C) Culdocentesis (D) Dilatation and curettage (E) Immediate appendectomy

A 60-year-old nulligravid woman comes to the physician because of intermittent, light vaginal bleeding for 4 months. She has no history of illness except for hypertension controlled with nifedipine therapy. Menopause occurred 9 years ago. Her last Pap smear was 2 years ago and showed normal findings. Her temperature is 37.1EC (98.8EF), pulse is 84/ min, and blood pressure is 138/86 mm Hg. There is a small amount of blood at the cervical os; examination is otherwise noncontributory. Which of the following is the most appropriate next step in management? (A) Reexamination in 6 months (B) Oral conjugated estrogen therapy (C) Colposcopy (D) Endometrial sampling (E) Ablation of the endometrium

A 41-year-old woman comes to the physician because of increasing menses over the past 3 years. Her menses occur at regular intervals and now last 12 days. Pelvic examination shows a firm, irregular pelvic mass. What is the most likely diagnosis? (A) Dermoid cyst (B) Ectopic pregnancy (C) Endometrioma (D) Leiomyomata uteri (E) Tubo-ovarian abscess

A 26-year-old woman comes to the physician for a routine health maintenance examination. Pelvic examination shows a 6-cm, cystic, adnexal mass. An x-ray of the abdomen shows calcifications. What is the most likely diagnosis? (A) Dermoid cyst (B) Ectopic pregnancy (C) Endometrioma (D) Leiomyomata uteri (E) Tubo-ovarian abscess

A 14-year-old girl is brought to the physician because she has not had a menstrual period for 5 months. Menarche was at 9 years of age. Menses have always occurred at increasing intervals. She had sexual intercourse with one partner 5 months ago. She is at the 50th percentile for height and 95th percentile for weight. Examination shows mild hirsutism with coarse hair on the upper lip and periareolar area. There are velvety-brown hyperpigmented lichenified plaques in the neck and axillary folds. Pelvic examination shows slightly enlarged ovaries bilaterally. Serum studies show: Glucose 250 mg/dl, Luteinizing hormone 30 mIU/mL, Follicle-stimulating hormone 9 mIU/mL, Dehydroepiandrosterone sulfate 3.7 mg/dL, (N=0.83.4), Thyroid-stimulating hormone 1 U/mL, Testosterone 105 mg/dL (N=2595), Prolactin 20 ng/mL. Which of the following is the most likely cause of this patient s amenorrhea? (A) Addison disease (B) Adult-onset 21-hydroxylase deficiency (C) Hyperandrogenic insulin-resistant acanthosis nigricans syndrome (D) Ovarian teratoma (E) Pregnancy

A 37-year-old woman, gravida 3, para 3, comes to the physician because of labor-like pains and a small amount of vaginal bleeding for 2 days. She has had increasingly heavy, regular menstrual periods over the past 3 years; her last menstrual period was 18 days ago. Her youngest child is 10 years old. The cervix is moderately effaced and 2 cm dilated; there is beefy red tissue in the os. Which of the following is the most likely diagnosis? (A) Carcinoma of the cervix (B) Carcinoma of the endometrium (C) Incomplete abortion (D) Pedunculated submucous leiomyoma (E) Sarcoma of the uterus

An 18-year-old nulligravid woman comes to the physician because of temperatures to 39oC (102.2oF), malaise, and multiple painful blisters on her vulva for 2 days; she has been unable to void for 12 hours. She reports that she has had unprotected sexual intercourse with a new partner several times over the past 3 weeks. She has bilateral inguinal adenopathy. There are shallow-based ulcers on the vulva, vaginal mucosa, and ectocervix. The most appropriate treatment is administration of which of the following? (A) Acyclovir (B) Ganciclovir (C) Immune globulin (D) Interferon (E) Zidovudine (AZT)

A 22-year-old woman, gravida 1, para 1, comes to the physician because of vaginal discharge and vulvar pruritus for 7 days. She is allergic to penicillin. Her last menstrual period was 1 week ago. There is a thin, bubbly, pale green discharge. A wet mount preparation shows a mobile, pear-shaped, flagellated organism. What is the most appropriate pharmacotherapy? (A) Ceftriaxone (B) Erythromycin (C) Metronidazole (D) Miconazole (E) Tetracycline

A 25-year-old woman, gravida 2, para 2, comes to the physician because of vaginal discharge, vulvar pruritus and burning, and dyspareunia for 3 days. She has no drug allergies. The vagina is tender and erythematous. A KOH wet mount preparation shows spores and hyphae. What is the most appropriate pharmacotherapy? (A) Ceftriaxone (B) Erythromycin (C) Metronidazole (D) Miconazole (E) Tetracycline

A 14-year-old girl is brought to the physician because of severe dysmenorrhea over the past year. The dysmenorrhea is accompanied by nausea and vomiting the first 2 days of her menstrual period and causes her to miss 2 days of school each month; aspirin does not relieve the pain. Menarche was at 11 years of age. She is sexually active with one partner and uses condoms for contraception. Pelvic examination shows no abnormalities. Serum studies show a luteinizing hormone concentration of 7 mIU/mL, folliclestimulating hormone concentration of 7 mIU/mL, and thyroid-stimulating hormone concentration of 4.0 U/mL. A pregnancy test is negative. Which of the following is the most appropriate next step in management? (A) Doxycycline therapy for 10 days (B) Acetaminophen therapy prior to expected menses (C) Ibuprofen therapy prior to expected menses (D) Codeine therapy during menses (E) Danazol therapy daily

A 24-year-old woman comes to the office for a gynecologic examination. This is her first visit and she has no complaints. She tells you that she has not had a Pap smear for several years. Menarche was at age 12 years and she has had normal menstrual cycles since then. She has had several sexual partners in the past but has been with her current partner in a monogamous relationship for 1 year. She reports that she had a chlamydial infection that was treated several years ago, but she denies a history of other sexually transmitted diseases. She has never been pregnant. On physical examination her cervix appears friable with a slight area of ulceration. There are several perineal and vaginal lesions that appear as small "cauliflower-like" projections. The results of the Pap smear, which return in 1 week, show mild dysplasia (LGSIL). Which of the following factors in this patient's history most closely correlates with the abnormal finding on Pap smear? (A) Condylomata acuminata (B) Condylomata lata (C) Early age at menarche (D) History of chlamydia (E) Nulliparity

Obstetrics

Which one of the following antepartum fetal assessment results provides reassurance of fetal well-being in a postdates pregnancy? A. B. C. D. E. A biophysical profile score of less than 4 A positive contraction stress test Decreased amniotic fluid volume noted on ultrasonography Spontaneous decelerations in fetal heart rate during a nonstress test A reactive nonstress test

A reactive nonstress test (normal fetal heart rate accelerations produced by fetal movements) provides reassurance of fetal well-being in a postdates gestation, using a simple, noninvasive method. Spontaneous decelerations of fetal heart rate have the same connotation as a nonreactive stress test: fetal compromise. Ultrasound examination is often combined with nonstress testing in postdates pregnancies. Oligohydramnios is common in postdates pregnancy and increases the risk of cord accidents, which are not predicted well by nonstress testing.

Which one of the following second-trimester findings would most strongly suggest the possibility of a twin gestation? A. B. C. D. Palpation of fetal parts A discrepancy between gestational age and uterine size Increased levels of chorionic gonadotropins in the urine The onset of polyhydramnios

If the uterus grows larger than expected during the second trimester one should always suspect, among other diagnoses, the presence of twins. Palpation of fetal parts is difficult before the third trimester. Gonadotropins are usually higher with twins, but the difference is not great enough for diagnostic usefulness. Polyhydramnios may occur with twins but may also suggest other diagnoses such as fetal malformations or maternal diabetes.

A 24-year-old white primigravida has developed several 1to 2-mm erythematous papules on her abdomen in the third trimester. They are pruritic and tend to appear in her striae. Liver function tests and a CBC are normal. Which one of the following is the most likely diagnosis? A. B. C. D. E. Pruritus gravidarum Spangler's papular dermatitis Impetigo herpetiformis Herpes gestationis Pruritic urticarial papules and plaques (PUPP)

The findings in this patient are most consistent with PUPP. This condition is usually benign, is not associated with increased fetal morbidity, and resolves after delivery, and there is usually no recurrence in subsequent pregnancies. Herpes gestationis, impetigo herpetiformis, and Spangler's papular dermatitis have different presentations and may be associated with increased fetal morbidity. Pruritus gravidarum is characterized by pruritus without skin lesions.

Which one of the following is the most reliable clinical predictor of gestational age? A. A pelvic examination performed before 12 weeks gestation by an experienced practitioner B. Date of first detected audible heart tones by electronic Doppler C. Date of first perception of fetal movement by the mother D. An accurately dated last menstrual period E. Fundal height measurement

All of the methods listed are useful in helping to predict gestational age. The most reliable clinical predictor, however, is an accurately dated last menstrual period.

Which one of the following is an absolute contraindication for a trial of vaginal birth after cesarean delivery (VBAC)? A. Previous cesarean delivery performed for failure of labor to progress, despite documented adequacy of labor B. Previous cesarean delivery via classic vertical uterine incision C. Previous cesarean delivery with a vertical abdominal wall incision and a low transverse uterine incision D. Two previous cesarean deliveries, both via low transverse uterine incisions E. Current term pregnancy without onset of labor 10 hours after spontaneous rupture of membranes
A previous cesarean delivery via classic vertical uterine incision is an absolute contraindication to a trial of vaginal birth after cesarean delivery (VBAC), due to a much higher risk of uterine rupture. VBAC success rates are quite good in women with a history of failure to progress in the previous labor. Spontaneous rupture of the membranes can still be managed with oxytocin in women with a previous cesarean delivery. A history of two previous low transverse cesarean sections is not a definite contraindication to VBAC. The type of abdominal wall incision has nothing to do with the risk of uterine rupture.

A 25-year-old primigravid woman at 42 weeks gestation delivers a 4000-g (8-lb 13-oz) newborn after induction of labor with oxytocin. The first and second stages of labor lasted 14 and 3 hours, respectively. A midline episiotomy was done, and the placenta appeared to be intact. Ten minutes after delivery, she has copious vaginal bleeding estimated to be 500 mL over a 5-minute period; the fundus is soft and boggy. Which of the following is the most likely cause of the hemorrhage? (A) Cervical laceration (B) Disseminated intravascular coagulation (C) Retained placental tissue (D) Uterine atony (E) Uterine inversion

A 24-year-old primigravid woman with type 1diabetes mellitus delivers a 3856-g (8-lb 8-oz) newborn at 38 weeks gestation. The pregnancy was complicated by poor control of her diabetes. The labor lasted 4 hours. Apgar scores were 7 and 7 at 1 and 5 minutes, respectively. Which of the following is the most appropriate neonatal blood test in the first 30 minutes after birth? (A) Determination of blood group and Rh (B) Measurement of hematocrit (C) Measurement of pH (D) Measurement of serum bilirubin concentration (E) Measurement of serum glucose concentration

A 20-year-old primigravid woman delivers a full-term 3200-g (7-lb 1-oz) newborn vaginally under epidural anesthesia. A second-degree midline episiotomy was required. On postpartum day 1, she has perineal pain. The perineum is slightly edematous with no evidence of purulent drainage or episiotomy breakdown. Which of the following is the most appropriate next step in management? (A) Sitz baths twice daily (B) Local injection of lidocaine in the perineum (C) Intravenous ampicillin therapy (D) Intramuscular morphine therapy (E) Epidural anesthesia

A 26-year-old woman, gravida 3, para 2, comes to the physician for her first prenatal visit at 36 weeks gestation. She has been taking over-the-counter ferrous sulfate. Although her intake of protein and calories has been adequate, she rarely eats fruits or fresh vegetables. This patient is at increased risk for a deficiency of which of the following? (A) Calcium (B) Folic acid (C) Vitamin A (D) Vitamin B12 (cobalamin) (E) Vitamin D

A 28-year-old woman who is 20 weeks pregnant with her third child comes to the office for a routine prenatal visit. She has two healthy children, ages 7 and 5 years. Her previous pregnancies were uncomplicated except for a cesarean delivery of the first child. She began using cocaine soon after the birth of her second child, and she has been in and out of drug treatment programs for the past 3 years. She has used crack cocaine on and off throughout this pregnancy. You have encouraged her to seek help; however, she has made no attempt to abstain from using cocaine and she refuses to commit herself to another drug treatment program. In your attempt to persuade this patient to stop using cocaine, you advise her that if she continues to use cocaine during the pregnancy, she increases her risk for which of the following? (A) Chorioamnionitis (B) Gestational diabetes (C) Placental abruption (D) Placenta previa (E) Preeclampsia

A 20-year-old woman who is 35 weeks pregnant with her first child is admitted to the hospital because of persistent hypertension and 1+ protein on urinalysis obtained 36 hours ago. She is confined to bed awaiting further diagnostic studies for preeclampsia. Her blood pressure is now 160/100 mm Hg. She is complaining of headaches, blurred vision and epigastric pain. At this time it is most appropriate to conclude that this patient has which of the following? (A) Chronic hypertension and requires antihypertensive therapy (B) Chronic hypertension, but no antihypertensive therapy is necessary (C) Mild preeclampsia and should continue bed rest pending further diagnostic studies (D) Severe preeclampsia and requires immediate medical management and delivery (E) Severe preeclampsia and should continue bed rest pending further diagnostic studies

Congestive Heart Failure

Which one of the following is true regarding medical therapy for chronic congestive heart failure (CHF)?
A. Antiarrhythmics should be used routinely for all patients B. Diuretics are more effective for treating symptoms related to low output than for treating symptoms of fluid overload C. Digoxin is most likely to be useful in patients with mild symptoms D. Vasodilators other than ACE inhibitors should be first-line therapy E. Angiotensin-converting enzyme (AC inhibitors often improve the symptoms of CHF

Angiotensin converting enzyme (ACE) inhibitors are playing an increasing role in the treatment of congestive heart failure (CHF); they are the vasodilators of choice for CHF, often improving both symptoms and longevity. Fewer patients respond to digoxin, and those most likely to respond have more severe symptoms. Diuretics are more useful for treating symptoms of fluid overload than for treating symptoms of low output. The usefulness of antiarrhythmics for severe CHF has not been determined.

All of the following drugs except ______ are effective in the treatment of systolic congestive heart failure.
A. B. C. D. E. digitalis Aldosterone antagonists Beta - blockers Alpha - blockers Angiotensin-converting enzyme inhibitors Beta-blockers and angiotensin-converting enzyme inhibitors significantly reduce mortality from systolic congestive heart failure. Digitalis has been shown to decrease diuretic dose and the requirement for recurrent hospitalization, while aldosterone antagonists are effective in patients with more advanced failure. Alpha-blockers have no current role in the management of congestive heart failure and could produce adverse effects secondary to reflex tachycardia associated with this class of drugs.

Depression

Which one of the following depressed patients is most likely to commit suicide?
A. A 26-year-old male who repeatedly denies any thoughts of suicide B. A 30-year-old female who has been hospitalized overnight on several occasions for attempted suicide C. A 50-year-old recently divorced alcoholic male who feels life is hopeless D. A 50-year-old female who thinks of suicide and fears she might act on her thoughts

Assessment of suicidal risk is critical in determining the need for and duration of hospitalization of depressed patients. Most suicides are planned, not impulsive, and carried out successfully most often by the elderly, males, those in poor health, alcoholics, schizophrenics, those who have recently lost a loved one (especially mate), and those suffering from depressive disorders. Many depressed patients think about suicide, and a physician should take these patients seriously; however, among this group of patients, the 50-year-old recently divorced alcoholic male has the highest risk of successful suicide.

Which one of the following is a normal aspect of a grief reaction? A. B. C. D. E. Anger and resentment over a loss Morbid preoccupation with feelings of worthlessness Marked psychomotor retardation Prolonged functional impairment Auditory hallucinations

Anger and resentment over a loss are part of a normal grief reaction. Prolonged functional impairment, marked psychomotor retardation, and morbid preoccupation with feelings of worthlessness suggest that a grief reaction is complicated by a major depression. Auditory hallucinations also suggest a comorbid condition such as psychosis.

A 48-year-old white female complains of a depressed mood, hypersomnia, and hyperphagia (especially for carbohydrates). Similar episodes have occurred during the fall and winter months for the past 5 years. Based on the above clinical history, which one of the following is the most likely diagnosis? A. B. C. D. E. Narcolepsy Bulimia Seasonal affective disorder Bipolar disorder Hypothyroidism

Seasonal affective disorder, or SAD, is an alteration of mood during those times of year when total light exposure is decreased, namely fall and winter. This condition causes symptoms of depression, including excessive somnolence and over-eating. Treatment with phototherapy, perhaps in addition to antidepressant medication, can be helpful.

A previously healthy 17-year-old girl is brought to the physician for evaluation because of loss of appetite, sleeplessness, and extreme irritability for 3 weeks. After missing many practices, she quit the softball team that she previously enjoyed. She often feels tired and has difficulty sitting still and concentrating on schoolwork. Her menses occur at regular intervals. She is 168 cm (5 ft 6 in) tall and weighs 50 kg (110 lb); BMI 18 kg/m2. Her pulse is 74/min, respirations are 16/min, and blood pressure is 110/70 mm Hg. Which of the following is the most likely diagnosis? (A) Adjustment disorder with mixed disturbance of emotions and conduct (B) Anorexia nervosa (C) Attention-deficit/hyperactivity disorder (D) Dysthymic disorder (E) Major depressive disorder

A 54-year-old woman comes to the physician 2 months after her husband died of a myocardial infarction. She tires easily and has chest pain when she inhales deeply at rest. She has had frequent crying spells associated with thoughts of her husband and feels uninvolved in daily activities. Which of the following is the most likely diagnosis? (A) Conversion disorder (B) Generalized anxiety disorder (C) Major depressive disorder (D) Normal grief (E) Pathologic grief

Which one of the following is needed to make a diagnosis of prementrual dysphoric disorder? A. B. C. D. E. A psychiatric referral because these are research-based criteria A family history of substance abuse in first-degree male relatives Daily diary of premenstrual symptoms for 2 consecutive months Symptoms that persist throughout pregnancy The patient s own belief that she has premenstrual physical or psychological problems

Women with severe affective symptoms that are relieved by menses often have premenstrual dysphoric disorder (PMDD). Irritability, tension, and dysphoria are the most prominent and consistently described symptoms. A primary care physician can use the research criteria from the DSM-VI to diagnose PMDD. For insurance purposes, the diagnosis is noted ay depression NOS because insurance companies do not recognize research criteria . All diagnostic criteria emphasize the periodicity and severity of symptoms, which means that a patient s subjective report is not enough to make the diagnosis. Until symptoms are confirmed by the patient s prospective daily ratings during at least two consecutive menstrual cycles, the physician can make the diagnosis provisionally. A negative impact on function and lifestyle is a key element in the diagnosis. The physician should instruct the patient to keep a premenstrual daily symptom diary for several consecutive months so that cycle-to-cycle variability can be examined. Based on their diaries, many women may be found to have a pattern of symptoms during the nonluteal phase, which is not characteristic of PMDD.

A woman brings in her 75 year old mother because she is concerned that her mother has become depressed after her stoke 6 months ago. The mother has not interested in former activities and has had a personality change. Which of the following statements is true about this patient s condition? A. Depression is common after a stroke. B. There is no rationale for treating stroke patients with antidepressants if they are at high risk for depression. C. Depression does not affect the patient s rehabilitation. D. Fluoxtine (Prozac) is the durg of choice for poststoke depression. E. A patient with difficulty folllowing three-step commands may have depression.
Depression affects 25-40% of patients within the first year after a stroke and is associated with an increased risk of poststroke mortality. It is unclear to what extent antidepressant treatment reduces this risk or which antidepressant is most efficacious for poststroke depression. Nevertheless, antidepressant medication should be considered for treating poststroke patient who are depressed or at significant risk for depression.

Which of the following statements about bipolar disorder is true? A. The hallmark is reliving a traumatic event in the form of nightmares. B. Hypersomnia occurs in the manic phase. C. Hyperphagia occurs in the manic phase. D. During a manic phase, the patient sleeps very little, yet seems to have a great deal of energy. E. Sleep disturbances are not always part of the manic phase.
Bipolar disorder is characterized by variations in mood from elation or irritability to depression. Manic episodes vary in severity and frequency from patient to patient. During a manic episode, sypmtoms include a decreased need for sleep, and elevated mood or euphoria, talkativeness, racing thoughts, increased sexual activity, and an increased optimism that ultimately can impair judgment. A single manic episode is enough for a diagnosis as long as the symptoms are not related to a general medical condition such as amphetamine abuse or pheochromocytoma. A patient with acute mania needs emergency psychiatric care. A diagnostic indicator is that these behaviors differ from the patient s usual personality and the episodic onset evolves gradually over weeks or months. Unfortunately, many patients are not accurately diagnosed for years, so gathering information on behavior and mood states from families can help. Medication is the key to stabilizing bipolar disorder. Mania is initially treated with a mood-stabilizing or an atypical antipsychotic agent or both, bearing in mind that these antipsychotic agents may increase the risk for changes in lipid levels, glucose abnormalities, and weight gain. Patients must regulate their sleep, because poor sleep or erratic hours may trigger mood disturbances.

Which of the following statements about depression in women is true? A. Depression affects women and men equally. B. Major depression affects 45% of adults in the U.S. each year. C. Hormonal fluctuation does not increase the risk for mood disorders. D. About 10% of childbearing women experience postpartum major depression. E. Antipsychotic agents should be used to treat postpartum major depression.
An all too common disorder, major depression affects 15-25% of adults in the U.S. each year, with women twice as likely as men to be affected. Because women are at increased risk for mood disorders, particularly during hormonal fluctuations (premenstrual and postpartum), researchers believe gonadal steroids play an integral role in the pathogenesis. About 10% of childbearing women have postpartum depression, which is a traumatic event with an enduring impact on their self-confidence as mothers and on infants social, emotional, and cognitive development. When the depression is persistent or severe enough to interfere with caring for herself or the baby, the mother should be evaluated for antidepressant treatment. A woman with psychosis must receive an emergency psychiatric evaluation.

All of the following except __________ may lead to treatment failure in depressed patients. A. B. C. D. E. Undiagnosed hypothyroidism Use of oral contaceptives Comorbid substance abuse Poor adherence Undiagnosed Alzheimer s disease

Some 10-40% of depressed patients taking an antidepressant are partially or totally resistant to the treatment. Depression can also recur while taking the medication. Several factors contribute to treatment failure, including undiagnosed or misdiagnosed medical conditions such as hypothyroidism and anemia. Undiagnosed neuropsychiatric disorders such as Alzheimer s disease can cause treatment failure. Comorbid disorders such as eating disorders or substance abuse may also affect treatment response. Psychotic depression and bipolar depression may require concurrent pharmacotherapy such as antipsychotic and mood stabilizing agents. Adverse effects of medication and poor adherence can be additional obstacles to successful treatment. But even with a thorough history and physical examination, good patient education, and close follow-up, some patients may still be resistant to treatment and should be referred for psychiatric consultation.

Abdominal Pain

Which one of the following is true regarding acute appendicitis in the elderly?
A. B. C. D. The fatality rate is the same as in younger patients A normal WBC count excludes the diagnosis The appendix is more likely to perforate in older patients Tenderness rarely localizes to the right lower quadrant

Although acute appendicitis is seen more frequently in younger patients, it produces greater morbidity and mortality in the elderly. Because of atypical presentations, delays in diagnosis, the presence of underlying disease, and the reluctance of doctors to operate on the elderly, perforation is more frequent in geriatric patients. The WBC count is usually elevated in acute appendicitis, but may be normal in up to 30% of elderly patients. Findings can be atypical on physical examination, but the majority of elderly patients have tenderness in the right lower quadrant. Death related to acute appendicitis in the elderly is most often from myocardial infarction, pulmonary embolus, or congestive heart failure. Although the case fatality rate has decreased, the likelihood of dying from acute appendicitis is still much higher in the elderly.

An 18-year-old woman comes to the emergency department because of lower abdominal pain for 16 hours. She also reports loss of appetite and mild nausea. She is sexually active and takes an oral contraceptive. Her last menstrual period was 3 weeks ago. Her temperature is 37.9EC (100.2EF). There is guarding and tenderness to palpation of the right lower quadrant of the abdomen; palpation of other areas of the abdomen results in referred pain in the right lower quadrant. Bowel sounds are absent. Pelvic examination shows scant mucous discharge from the cervical os. Palpation on the right produces pain. There is no cervical motion tenderness, and the adnexa and ovaries appear normal. Which of the following is the most likely diagnosis? (A) Appendicitis (B) Ectopic pregnancy (C) Ovarian cyst (D) Tubo-ovarian abscess (E) Ureteral calculus

A 49-year-old man with alcoholism comes to the emergency department because of progressively severe abdominal pain over the past 2 days; the pain now radiates to the back. He also has nausea and vomiting. Examination shows voluntary guarding in the upper abdomen with tenderness to percussion. Hematocrit is 53%, leukocyte count is 12,000/mm3, and serum amylase activity is 700 U/L. Which of the following is the most appropriate next step in management? (A) Observe in the emergency department (B) Send home with analgesic therapy (C) Send home with antibiotic therapy (D) Admit to the hospital for endoscopy (E) Admit to the hospital for medical management (F) Admit to the hospital for an operation

A 62-year-old woman comes to the physician because of pain in the left lower quadrant of the abdomen for the past 3 days. She had similar episodes on three previous occasions over the past 6 months. Examination shows abdominal tenderness in the left lower quadrant with rebound tenderness. Her hemoglobin concentration is 13.6 g/dL and leukocyte count is 15,400/mm3 with 82% segmented neutrophils and 18% lymphocytes. Urinalysis shows 56 WBC/hpf. Which of the following is the most likely diagnosis? (A) Acute cholecystitis (B) Acute diverticulitis (C) Acute pyelonephritis (D) Appendicitis (E) Spontaneous bacterial peritonitis

A 40-year-old woman comes to the emergency department because of fever and increasingly severe pain in the right upper quadrant of the abdomen for 18 hours. She has had no nausea or vomiting. There is no history of fatty food intolerance or previous symptoms. The patient returned from a trip to Mexico 6 months ago. Her temperature is 38.9EC (102EF), pulse is 110/min, respirations are 18/min, and blood pressure is 130/85 mm Hg. Examination shows decreased breath sounds over the right base. Abdominal examination shows tenderness to percussion over the right upper quadrant and normal bowel sounds; Murphy sign is absent. Leukocyte count is 20,500/mm3 (25% bands, 55% eosinophils, 5% lymphocytes, and 15% monocytes). Which of the following is the most likely diagnosis? (A) Amebic abscess (B) Appendicitis (C) Cholecystitis (D) Hepatitis (E) Pyogenic abscess

A 46-year-old man comes to the physician because of intermittent lower abdominal pain over the past 3 months. There is no family history of cancer. Examination shows no other abnormalities. His hematocrit is 38%. Test of the stool for occult blood is positive. Colon contrast studies show a 1.5-cm polyp in the descending colon. An upper gastrointestinal series shows no abnormalities. Which of the following is the most appropriate next step in management? (A) Reexamination in 1 year (B) Repeat test of the stool for occult blood after 3 days on a meat-free diet (C) Measurement of serum carcinoembryonic antigen (CEA) concentration (D) Colonoscopy with polypectomy (E) Total colectomy

A 75-year-old woman comes to the clinic because she has band-like burning pain in the right upper quadrant extending from the epigastrium around to the midline of the back. On physical examination, there is no abdominal tenderness. Findings on ultrasonography of the gallbladder are normal; serum amylase concentration is normal. Which of the following is the most likely diagnosis? (A) Acalculous cholecystitis (B) Chronic relapsing pancreatitis (C) Diverticulitis of the cecum (D) Herpes zoster (E) Penetrating duodenal ulcer

A 68-year-old woman comes to the office for flexible sigmoidoscopy as part of a yearly screening. A 3-cm polyp is found in the sigmoid colon and is removed. She returns now to the office, 6 hours later, complaining of left lower quadrant pain, fever, nausea and vomiting. Vital signs are: temperature 38.1C (100.6F), pulse 110/min, respirations 26/min and blood pressure 120/60 mm Hg. Abdominal examination discloses bowel sounds, tenderness and guarding in the left lower quadrant. Rectal examination shows no stool and only tenderness superiorly. Which of the following is the most appropriate next step? (A) Obtain an angiogram to rule out intestinal ischemia (B) Obtain immediate consultation with a surgeon (C) Pass a soft rubber rectal tube under fluoroscopy (D) Repeat the flexible sigmoidoscopy in order to evaluate the operative site (E) Start hydrocortisone, intravenously, to decrease any inflammatory response

A 57-year-old woman is admitted to the hospital for evaluation of nausea, vomiting, crampy abdominal pain and abdominal distention. Her medical history includes cholecystectomy and appendectomy 5 years ago. X-ray film of the abdomen is shown. Which of the following is the most likely cause of her symptoms? (A) Adhesive band (B) Femoral hernia (C) Gallstone ileus (D) Perforated diverticulum (E) Sigmoid colon carcinoma

Hypertension

A 36-year old woman, with no significant past medical history presents for heal care maintenance. On this visit, her blood pressure is 144/94. She has not family history of HTN and no prior elevations of BP noted. Which of the following next steps would be least appropriate? A. Diagnose HTN and begin nonpharmacologic therapies. B. Repeat BP measurement after the patient has been sitting quietly for 5 minutes. C. Schedule a follow-up visit to recheck BP. D. Examine patient for evidence of HTN changes (eg. Retinopathy) E. Order an ambulatory BP monitor.
JNC 7 recommends that patients should be seated quietly for a least 5 minutes in a chair rather than on an exam table, with feet on the floor and arm supported at heart level for measurement of BP. BP should be determined by the mean of at least 2 readings. Furthermore, experts commonly recommend that clinicians diagnose HTN only after obtaining 2 or more elevated readings at 2 or more office visits at intervals of 1 to several weeks.

Which of the following statements regarding benefits of treating HTN is true? A. Treatment of HTN reduces stroke rates but does not significantly reduce coronary heart disease (CHD) rates. B. Treatment of HTN reduces CHD rates but does not significantly reduce stroke rates. C. Treatment of HTN reduces both stoke rates and CHD rates. D. Treatment of HTN reduces stroke rates but increases CHD rates. E. None of the above.

Treatment of HTN reduces both stoke and coronary heart disease rates.

A 52-year old Hispanic man with long-standing HTN presents for routine follow-up. At this visit, his BP is 172/94. On questioning, the patient admits to nonadherence to his medications. All of the following factors are related to patient adherence except: A. B. C. D. E. Complexity of medication regimen Patient gender Medication costs Side-effect profile Multiple daily doses

The complexity of the management regimen is related to patient adherence, as is the number of daily doses required. Studies have shown that HTN patients asked to take only 1 pill per day are more adherent to the drug regimen than patients having to take medications twice per day. Cost and side-effect profile have also been found to affect adherence. The patients gender has not been found to affect adherence rates.

At a recent visit, her main complaint is new-onset peripheral edema. Which of the following medications has the highest incidence of this adverse effect? A. B. C. D. E. spironolactone hydrochlorothiazide enalopril metoprolol amlodipine

Amlodipine belongs to a class of medications call CCBs. These medications block the transport of calcium into the smooth muscle cells lining arteries. Because calcium is required for muscle contraction, blocking calcium transport relaxes artery muscles and thus dilates the arteries. CCBs also blunt postural skin vasocontriction, an autoregulatory mechanism that minimized gravitational increases in capillary pressure and avoids fluid extravasation when standing. Because of this effect, edema is relatively common adverse effect of amlodipine use, with an incidence of approximately 22%.

This patient presents with pain in her left great toe. You examine her and diagnose an acute attack of gout. Which of the following medications has the highest incidence of this adverse effect? A. B. C. D. E. hydrocholorthiazide spironolactone enalopril metoprolol amlodipine

Although serum uric acid levels can increase during HCTZ treatment, precipitation of acute gout is rare. This can be limited by maintaining adequate hydration.

A 45-year old man who was recently stared on an ARB returns for a follow-up visit. You want to check his creatinine level to ensure that he is tolerating the medication. His baseline Cr was 1.0 mg/dL. How much of an increase I n creatinine level secondary to ACEI or ARB is considered acceptable? A. B. C. D. E. 0.5 mg/dL 30% 50% 0.01 mg/dL A doubling of serum Cr level

A limited increase in serum creatinine of as much as 30% above baseline with ACE inhibitors or ARBs is acceptable and not a reason to withhold treatment unless hyperkalemia develops.

A 54-year old woman with newly diagnosed HTN asks you about nonpharmacologic treatments to reduce her BP. Which of the following interventions as not been proved to reduce BP? A. B. C. D. E. F. Weight reduction Adoption of the Dietary Approaches to Stop HTN (DASH) diet Dietary sodium reduction Physical activity Moderation of alcohol consumption Limitation of dietary carbohydrates

All of the interventions listed except for limitation of dietary carbohydrates have been associated with BP reductions. According to JNC 7, the following BP reductions in response to lifestyle modifications can be expected: Weight reduction 5-20 mm Hg/10 kg loss DASH diet 8-14 mm Hg Sodium reduction 2-8 mm Hg Physical activity 4-9 mm Hg Mod ETOH cons 2-4 mm Hg

The most common surgically curable cause of hypertension in the United States is which one of the following? A. B. C. D. E. Primary aldosteronism Pheochromocytoma Cushing's syndrome Renal artery stenosis Coarctation of the aorta

Partial obstruction of the renal artery by intrinsic or extrinsic disease is responsible for only 1% to 2% of cases of hypertension. Nevertheless, it represents the most common curable cause of this widespread disease (excluding oral contraceptives). Because of high cost and low positive yield, formal evaluation of renal artery stenosis is recommended only for patients in whom the onset of hypertension has occurred before the age of 30 years, those with laboratory evidence of secondary hyperaldosteronism, those with a bruit, or those in whom hypertension is severe and responds poorly to medical therapy. Positive identification is accomplished via renal arteriography and renal vein determinations. Primary aldosteronism, pheochromocytoma, Cushing's syndrome, and coarctation of the aorta are all curable causes of hypertension, but are all less prevalent than renovascular disease.

A 50-year-old white male with a history of type 1 diabetes mellitus and microalbuminuria is found to have hypertension. The preferred antihypertensive medication would be A. B. C. D. E. captopril (Capoten) hydrochlorothiazide (HydroDIURIL) terazosin (Hytrin) propranolol (Inderal) nifedipine (Procardia)

ACE inhibitors, such as captopril, reduce proteinuria and slow the progression of renal disease in patients with diabetes. Hydrochlorothiazide inhibits insulin secretion and may increase blood lipids. Propranolol can cause these same effects and also blocks the adrenergic symptoms of hypoglycemia. Data on the effect of nifedipine on microalbuminuria are unclear. No information is available regarding terazosin in diabetes.

Which of the following is classified as stage 1 hypertension, according to the JNC VII report? A. A systolic blood pressure (SBP) less than 120 mm Hg and a diastolic blood pressure (DBP) less than 80 mm Hg. B. An SBP of 160 mm Hg or higher or a DBP of 100 mm Hg or higher. C. A DBP between 90 and 99 mm Hg. D. An SBP between 120 and 139 mm Hg. E. A DBP between 80 and 89 mm Hg.

Which one of the following statements regarding drugs that affect blood pressure is true? A. The use of thiazide diuretics may cause hypoglycemia. B. Angiotensin-converting enzyme (ACE) inhibitors are considered firstline therapy for patients who have both diabetes and hypertension. C. The high incidence of side effects caused by angiotensin II receptor blockers is a major barrier to adherence. D. The use of beta blockers is not recommended for patients who have had a myocardial infarction (MI). E. Trials of calcium channel blockers (CCBs) have shown unequivocally that CCB monotherapy should be recommended as first-line treatment for hypertension.

Which one of the following would be most likely to have secondary hypertension? A. A 39-year-old white male who weighs 119 kg (262 lb) and whose blood pressure is 142/94 mm Hg B. A 48-year-old African-American female with left ventricular hypertrophy on echocardiography whose blood pressure is 162/98 mm Hg C. A 62-year-old African-American male with a strong family history of hypertension D. A 78-year-old white female with abdominal bruits whose blood pressure is 182/102 mm Hg Physical findings which suggest secondary hypertension include the presence of abdominal bruits, particularly those that lateralize or have a diastolic component. Excess body weight is correlated closely with increased blood pressure, but is not a cause of secondary hypertension. Hypertension is the most important risk factor for stroke, but a history of stroke is not an indication of secondary hypertension. Left ventricular hypertrophy is a result of hypertension, but is not an indication of secondary hypertension. The prevalence of hypertension is greater in African-Americans than in whites, but African-American race is not a risk factor for secondary hypertension.

Skin Lesions

A 32-year-old farmer comes to your office because of an upper respiratory infection. While he is there he points out a lesion on his forearm that he first noted approximately 1 year ago. It is a 1-cm asymmetric nodule with an irregular border and variations in color from black to blue. The patient says that it itches and has been enlarging for the past 2 months. He says he is so busy that he is not sure when he can return to have it taken care of. In such cases the best approach would be to A. freeze the site with liquid nitrogen B. perform an elliptical excision as soon as possible C. use electrocautery to destroy the lesion and the surrounding tissue D. perform a shave biopsy, with a recheck in 2 months for signs of recurrence E. perform a punch biopsy and have the patient return if the biopsy indicates pathology Despite this individual's busy schedule, he has a potentially life-threatening problem that needs proper diagnosis and treatment. Though an excisional biopsy takes longer, it is the procedure of choice when melanoma is suspected. After removal and diagnosis, prompt referral is essential for further evaluation and therapy. A shave biopsy should never be done for suspected melanoma, as this is likely to transect the lesion and destroy evidence concerning its depth, thus making it difficult to assess the prognosis. A punch biopsy should be used only with discretion in suspected melanoma, when the lesion is too large for complete excision, or if substantial disfigurement would occur. Since this may not actually retrieve cancerous tissue from an unsampled area of a large lesion that might be malignant, it would be safest to refer such a patient. Neither cryotherapy nor electrocautery should be used for a suspected melanoma.

A 65-year-old white male comes to your office with a 0.5-cm nodule which has developed on his right forearm over the past 4 weeks. The lesion is dome shaped and has a central plug. You schedule a biopsy but he does not return to your office for 1 year. At that time the lesion appears to have healed spontaneously. The most likely diagnosis is A. B. C. D. E. benign lentigo lentigo maligna basal cell carcinoma squamous cell carcinoma keratoacanthoma

Keratoacanthoma grows rapidly and may heal within 6 months to a year. Squamous cell carcinoma may appear grossly and histologically similar to keratoacanthoma but does not heal spontaneously. The other lesions do not resemble keratoacanthoma.

A 67-year-old man comes to the physician because of a nontender lesion on the top of his forehead that has recently bled. He states that the lesion has been there since he was hit on the head with a bottle 18 months ago. Examination shows erythematous areas on other parts of his forehead. The lesion is 1 cm in diameter with a rolled-up, darkened border and a translucent, ulcerated center. Which of the following is the most likely diagnosis? (A) Actinic keratosis (B) Basal cell epithelioma (C) Kaposi sarcoma (D) Melanoma (E) Seborrheic keratosis

The lesion shown is of a 1-year-old girl who is brought to the office by her parents because they want a second opinion regarding the growth on her leg. The mother notes that it was small and flat at birth and has grown thicker in the past 6 months. The mother says, "My usual physician seems unconcerned, but my mother-in-law says it looks awful and that something should be done." The child is otherwise healthy. At this time it is most appropriate to tell the parents that the best cosmetic result will occur with which of the following? (A) Injection with a corticosteroid (B) Observation only (C) Radiation therapy (D) Surgical excision (E) Topical application of a corticosteroid

A 58-year-old man comes to the office because of a lesion on the lip, which is shown. The patient says he has had the lesion for about 9 months. He has not seen a physician for 5 years and he is in the office today only because, he says, "My wife made me come." On physical examination the lower lip is fixed to the anterior aspect of the mandible. Which of the following is the most likely diagnosis? (A) Basal cell carcinoma (B) Keratoacanthoma (C) Leukoplakia (D) Melanoma (E) Squamous cell carcinoma

Diabetes

Which one of the following is commonly associated with diabetes mellitus? A. B. C. D. E. Candida vaginitis Amebic hepatic abscess Hirsutism Amaurosis fugax Seborrheic dermatitis

Fungal infections of the vulva and vagina are a common problem for women who have diabetes, resulting in an abnormal, malodorous discharge, and irritation of the tissues. There is no association between the other conditions listed and diabetes.

A 40-year-old female comes to your office for a routine examination. She has been in good health and has no complaints other than obesity. Her mother is diabetic and the patient has had a child that weighed 9 lb at birth. Her examination is negative except for her obesity. A fasting glucose level is 128 mg/ dL, and when repeated 2 days later it is 135 mg/dL. In this case proper management would be to A. B. C. D. tell the patient that she has impaired glucose homeostasis but is not diabetic order a glucose tolerance test begin an oral hypoglycemic agent diagnose type 2 diabetes mellitus and begin diet and exercise therapy

The new criteria for diagnosing diabetes mellitus include any one of the following findings on two different days: symptoms of diabetes (polyuria, polydipsia, weight loss) plus a casual (without regard to time since last meal) glucose level >200 mg/dL; a fasting plasma glucose level >126 mg/dL; or a 2-hour postprandial glucose level >200 mg/dL after a 75-gram glucose load. The criteria for impaired glucose homeostasis include either a fasting glucose level of 110-125 mg/dL (impaired fasting glucose) or a 2-hour postprandial glucose level of 140-199 mg/dL. Normal values are now considered <110 mg/dL for fasting glucose and <140 mg/dL for 2-hour postprandial glucose. The oral glucose tolerance test is no longer recommended.

Otitis Media

A mother brings a 3-year old boy to your clinic. She says that the child has been crying and pulling at his left ear for the last 2 days. She thinks he had a fever but did not have a thermometer and did not take his temperature. Although the child is crying, his mother can console him, and he appears well. Which one of the following findings is most specific for acute otitis media? A. Fluid level behind tympanic membrane B. Loss of light reflex C. Bulging tympanic membrane D. Erythema of the ear canal E. Decreased movement of tympanic membrane with insufflation.

Erythema of the external ear canal is more consistent with otitis externa than otitis media. While a bulging tympanic membrane with or without fluid behind it and loss of the light reflex may be sings of acute otitis media, the most sensitive finding is decreased mobility of the tympanic membrane with insufflation.

A 24-month-old African-American female whom you have followed for routine well child care and a few episodes of otitis media is brought to the office by her mother for a regular well child visit. The mother is concerned that the child's language development seems to be slower than she remembers with her older children. Which one of the following would be a cause for concern at this age? A. She is making sentences of only two or three words B. She is unable to name pictures on a standardized test, such as the Denver Developmental Screening Test C. She is unable to correctly recognize three of four colors D. She is unable to give her first and last name E. Her total vocabulary includes about 40 words

Since language development is most rapid at about the age of 2 years, it is very important to be sensitive to clues of delayed development. There are a variety of sources for such milestones, including the Denver Developmental Screening Test. According to the guidelines used for this test, as well as those in standard references, all the behaviors listed are within the normal range, with the exception of the inability to name pictures on a standardized test.

A 2-year-old white male has had four episodes of otitis media within the last 6 months, all of which completely cleared after 10 days of appropriate antimicrobial therapy. You see him in your office following his most recent episode. Physical examination is completely normal at this time. You decide to attempt to reduce the number of recurrences. Of the following, the best initial intervention is to A. schedule myringotomy and insertion of ventilating tubes B. perform an allergy survey and consider allergy hyposensitization injections C. prescribe a course of oral steroids to decrease swelling of the eustachian tube mucosa D. prescribe daily amoxicillin or sulfisoxazole, especially during winter months E. schedule adenoidectomy
Prophylactic antibiotics are considered the most effective initial therapy in the prevention of recurrent acute otitis media. Myringotomy and ventilating tubes may also be effective, but should be reserved for patients who fail to respond to antimicrobial prophylaxis or in whom it is contraindicated. Adenoidectomy and allergy surveys are generally reserved for patients who require multiple myringotomies and tubes or who have other indications for these interventions. Topical and oral steroids have been used for persistent middle ear effusion with unproven results and are generally avoided;

they are not usually considered in the absence of effusion.

A previously healthy 3-month-old boy is brought to the physician by his parents because he continues to have fever and ear pain despite treatment with amoxicillin for 72 hours. Examination shows an immobile, red, and opaque tympanic membrane. Which of the following is the most likely pathogen? (A) Chlamydia trachomatis (B) Group A streptococcus (C) Haemophilus influenzae (D) Mycoplasma pneumoniae (E) Staphylococcus aureus

Obesity

Which one of the following statements regarding childhood obesity is true? A. Results of long-term treatment have generally been good B. Weight loss at any age beyond 1 year results in a decreased number of adipose cells C. Thyroid function is frequently found to be decreased in obese children D. Childhood obesity is not a reliable predictor of adult obesity Weight loss at any age beyond 1 year decreases the size of adipose cells, but the total number of cells remains the same. There is no evidence of decreased thyroid function in uncomplicated obesity. No defective peripheral utilization of thyroid hormone is noted. The use of thyroid hormones to treat obesity of nonthyroid origin is inappropriate. The long-term management of obesity in children, as in adults, has generally ended in failure. The decision to undertake treatment of an obese child rests on many factors, the most important being the degree of patient and parent cooperation, and this is not easy to obtain. Studies have shown that a minority (10% to 30%) of obese adults were obese as children. Thus, it is not a direct predictor of adult obesity; the probability of a nonobese child becoming an obese adult decreases with a greater time interval between the onset of obesity and adulthood, but increases with severity of childhood obesity, onset in adolescence, and a family history of obesity.

Rashes

The most appropriate therapy to clear up poison ivy on the forearm of a 3-year-old child is A. B. C. D. topical fluorinated steroids oral prednisone in tapered doses oral antihistamines for 10-14 days oral ephedrine for 1 week

The treatment of choice for contact dermatitis is fluorinated steroids. For children particularly, topical therapy is most appropriate, although some absorption will take place. While antihistamines will relieve some of the pruritis, they will not resolve the allergic response any quicker. Oral ephedrine is basically ineffective in dermatitis.

A mother brings her 7-year old daughter to the office and says she has had 3 days of a generalized erythematous rash on her face and arms. Her temp is 1010F. The mother also says the rash became more severe last night after the child took a bath. Otherwise, she does not appear acutely ill. What is the most likely diagnosis? A. Scarlet fever B. meales C. roseola D. Fifth disease (parvovirus B19) E. Enterovirus infection
The child has a typical presentation for parvovirus B19 infection. The rash associated with this disease is usually seen on the cheeks and is called a slapped-cheek appearance. In our current vaccine era measles would be uncommon, and affected children are much sicker. The rash with scarlet fever caused by group A strep is the consistency of sand paper and involves the volar surfaces of the arm. This illness would typically be accompanied by pharyngitis and cervical lymphadenopathy. Enterovirus infections end to occur in younger children and present with fever and generalized erythematous macular rash. This rash, unlike the B19 rash is neither heat nor temperature sensitive.

A 56-year-old man has had the painful weeping rash shown for 2 days. He underwent chemotherapy for non-Hodgkin lymphoma 1 year ago. His temperature is 36.7EC (98EF), pulse is 80/min, and blood pressure is 138/76 mm Hg. Examination shows no other abnormalities. Which of the following is the most likely diagnosis? (A) Herpes zoster (B) Impetigo (C) Pyoderma gangrenosum (D) Syphilis (E) Systemic lupus erythematosus

A 67-year old man comes to the office with a painful vesicular rash that covers the right side of his chest. He tells you the rash appeared a few days ago. You suspect he has herpes zoster infection. Which one of the following statements about this condition is false? A. Caused by a reactivation of varicella zoster virus. B. The patient is no longer contagious at this stage. C. Antiviral therapy should be administered within 72 hours D. Dissemination of the infection may occur in immunocompromised patients E. Corticosteroids have not proven beneficial in treating this disorder.
About 20% of adults will experience recurrence of primary varicella zoster virus infection, usually at an advanced age. The virus is reactivated from the dorsal root ganglion, replicates, and travels down sensory nerves. A vesicular rash ensues, typically following a dermatomal distribution. Patients with the virus can transmit infection to those not previously exposed or not vaccinated against it. Dissemination and recurrence are common in immunocompromised patients. Treatment with oral antiviral agents should be started within 72 hours as this is the period of most active viral replication. Corticosteroids have not proven beneficial.

17-year-old girl has had pruritus and an increasingly severe rash. She has no history of skin problems or associated symptoms. She takes no medications. Her sister with whom she shares a room had a similar condition during the previous week. The patient s temperature is 36.8EC (98.2EF). There are multiple 2- to 5-mm erythematous papules over the trunk, especially at the waistline, and over the forearms, hands, and fingers. There is no lymphadenopathy or hepatosplenomegaly. Which of the following is the most likely causal organism? (A) Epstein-Barr virus (B) Group A streptococcus (C) Measles virus (D) Sarcoptes scabiei (E) Varicella-zoster virus

A 30-year-old woman comes to the physician because of an itchy, scaly rash for 1 year; the rash is most severe over her elbows and knees. The rash occurs only in the winter months and is moderately relieved by emollients. Examination shows discrete oval plaques 46 cm in diameter over the knees and elbows; they have an erythematous base and the overlying scale is silvery. What is the most likely diagnosis? (A) Atopic dermatitis (B) Dyshidrotic eczema (C) Irritant contact dermatitis (D) Nummular eczema (E) Psoriasis

92. A 23-year-old woman comes to the physician because of pale spots on her back for the past 2 months. She first noted a 3 x 4-cm oval patch on her upper back, followed by smaller spots with occasional itching. She has tried tanning oils and salons with no relief. Examination shows multiple 3- to 5mm macules on her back in a Christmas-tree distribution; the macules are paler than the surrounding skin. (A) Atopic dermatitis (B) Irritant contact dermatitis (C) Nummular eczema (D) Pityriasis rosea (E) Tinea corporis

A 17-year-old boy has had a rash involving the face, neck, upper chest, and upper and lower extremities for 5 days. During a recent trip to Mexico, he went scuba diving and encountered many jellyfish. He also had diarrhea treated with trimethoprim-sulfamethoxazole. His temperature is 37.8EC (100EF). Examination shows a confluent, erythematous, pruritic rash; no scaling or vesicles are noted. Examination is otherwise unremarkable. Which of the following is the most likely cause of this condition? (A) Eczema (B) Photosensitivity (C) Reaction to a jellyfish sting (D) Salmonella typhi (E) Toxicogenic Escherichia coli

Hyperlipidemia

A 12-month-old boy is brought to the office by his mother for his routine health check-up. She informs you that she has just been diagnosed with hypercholesterolemia. Her fasting serum total cholesterol concentration was 260 mg/dL and her LDL-cholesterol concentration was 130 mg/dL (rec<129 mg/dL). She is unaware of a family history of coronary artery disease because she was raised by her godmother when her parents died in their early 30s in a motor vehicle accident. A special diet has been recommended for her; however, she is very concerned about the risk of hypercholesterolemia for her son. Which of the following is the most appropriate management at this time? (A) Ask her to reduce the child's fat intake and give him skim milk instead of whole milk (B) Do nothing until the child is 2 years of age (C) Have the child return for a fasting lipoprotein analysis (D) Obtain a random serum total cholesterol concentration for the child today (E) Refer the child to a lipid specialist

Acute URI

A 32-year-old man has had progressive weakness in his arms and legs over the past 4 days. He has been well except for an upper respiratory tract infection 10 days ago. His temperature is 37.8EC (100EF), pulse is 94/min, respirations are 42/min and shallow, and blood pressure is 130/80 mm Hg. There is symmetric weakness of both sides of the face and of the proximal and distal muscles of the upper and lower extremities. Sensation is intact. No deep tendon reflexes can be elicited; the plantar responses are flexor. Which of the following is the most likely diagnosis? (A) Acute disseminated encephalomyelitis (B) Guillain-Barr syndrome (C) Myasthenia gravis (D) Poliomyelitis (E) Polymyositis

A 19-year-old man has had general malaise, recurrent sore throats, anorexia, swollen glands, and a 4-kg (9-lb) weight loss over the past 5 weeks. Four weeks ago, a throat culture grew Streptococcus pyogenes (group A), and the patient was treated with penicillin. His temperature is 38.5EC (101.3EF). Examination shows exudative pharyngitis, anterior and posterior cervical lymphadenopathy, and splenomegaly. Laboratory studies show: Hematocrit 37%, Leukocyte count 3200/mm3, Segmented neutrophils 55%, Lymphocytes 34%, Platelet count 84,000/mm3, Serum AST 75 U/L Which of the following is the most appropriate next step in diagnosis? (A) Antiplatelet antibody studies (B) Antistreptolysin O titer (C) Bone marrow examination (D) Screening for hepatitis (E) Serologic test for Epstein-Barr virus

A 79-year-old woman who is in a chronic care facility has fever, headache, sore throat, myalgias, and a nonproductive cough. Her temperature is 39.2EC (102.6EF), pulse is 104/min, respirations are 22/min, and blood pressure is 136/76 mm Hg. She appears ill. Her leukocyte count is 6200/mm3 (54% segmented neutrophils, 1% bands, 3% eosinophils, 2% basophils, 28% lymphocytes, and 12% monocytes). An x-ray of the chest shows no abnormalities. Immunization with which of the following vaccines is most likely to have prevented this condition? (A) BCG (B) Haemophilus influenzae type b (C) Influenza (D) Pertussis (E) Pneumococcal

Headache

A 43-year-old man comes to the emergency department at 3:00 AM because of a constant severe headache for 1 hour. The pain is localized behind the left eye. He also has had a watery discharge from the left nostril. He has had similar episodes nightly over the past week. He has a history of similar symptoms over a 3-week period 2 years ago. His left pupil is smaller than his right, and there is ptosis on the left. A watery discharge is visible in the left naris. What is the most likely diagnosis? (A) Acute sinusitis (B) Subarachnoid hemorrhage (C) Cluster headache (D) Intracranial tumor

A 19-year-old woman comes to the physician because of recurrent severe headaches for 1 year. The headaches areunilateral and throbbing and are accompanied by nausea, vomiting, and light sensitivity. The headaches occur once or twice monthly, reach their peak intensity in 1 hour, and last 1224 hours. There is no aura. Examination shows no abnormalities. What is the most likely diagnosis? (A) Cluster headache (B) Intracranial tumor (C) Migraine (D) Subarachnoid hemorrhage

A moderately obese 14-year-old girl has a 2-week history of severe bifrontal headaches and early morning vomiting. She appears alert and is cooperative. She is right-handed. Her pulse is 82/ min, and blood pressure is 112/76 mm Hg. Funduscopy results are shown. Her visual acuity is 20/20 bilaterally; neurologic examination and CT scan of the head show no abnormalities. Which of the following is the most likely diagnosis? (A) Migraine (B) Optic neuritis (C) Posterior fossa tumor (D) Pseudotumor cerebri (E) Tension headaches (E) Lumbar puncture

A 38-year-old nurse comes to the emergency department after leaving work early because of a "horrible headache." She has had a "cold" with sinus congestion for the past week, and yesterday she began taking an over-thecounter combination of diphenhydramine and pseudoephedrine. She tells you she has a history of "migraines," multiple allergies, premenstrual syndrome and depression, for which she takes phenelzine (a monoamine oxidase inhibitor). Vital signs are: temperature 37.2C (99.0F), pulse 90/ min, respirations 16/min and blood pressure 210/118 mm Hg. Which of the following is the most appropriate action? (A) Administer meperidine, intramuscularly (B) Administer phentolamine, intravenously (C) Order CT scan of the head (D) Order transillumination of the sinuses (E) Prescribe oral oxycodone and nasal corticosteroids

Allergic Rhinitis

Asthma

Which of the following is/are components of asthma? A. Reversible airway obstruction. B. Hyperreactivity of the airway. C. Inflammation of the airway. D. None of the above. E. All of the above.

A quick-relief medication used in the management of asthma is _____. a) inhaled albuterol b) inhaled salmeterol c) oral montelukast d) inhaled fluticasone Albuterol is considered a quick-relief medication. It is a beta2-adrenergic agent usually given by inhalation. Salmeterol, montelukast, and fluticasone are long-term controller medications.

Which of the following asthma controller agents does not provide an anti-inflammatory effect? a) ICS b) Inhaled salmeterol c) Oral montelukast d) Omalizumab e) Nedocromil Long-acting beta2-agonists (eg, salmeterol) should not be used as a single controller agent because they exert little, if any, anti-inflammatory effect. ICS medications; LTRAs, such as montelukast; mast cell stabilizers, such as nedocromil and cromolyn; and anti-immunoglobulin E (anti-IgE) antibodies all provide an anti-inflammatory effect.

A 45-year-old woman with a history of lifelong asthma presents to you for follow-up of her blood pressure. Despite losing 10 lb and changing her diet, her blood pressure remains elevated. Which class of blood pressure medication could exacerbate her asthma and therefore should be avoided? a) Thiazide diuretics b) Calcium channel blockers c) Beta blockers d) Angiotensin-converting enzyme inhibitors Beta blockers can cause obstruction of the airways in patients with asthma and should be avoided. Even the selective beta blocker (beta1) can have this effect, especially at the higher doses. The local use of beta1 blockers to treat glaucoma has caused a worsening of asthma.

Mast cells are the major pulmonary source of which chemical mediator of the acute asthmatic response? a) Acetylcholine b) Histamine c) Bradykinin d) Nitric oxide

Mast cells are the major pulmonary source of histamine. Acetylcholine is derived from the intrapulmonary motor nerves, causing vasoconstriction. Bradykinin is derived from plasma precursors. Nitric oxide is released by airway epithelial cells.

A 27-year-old man with a severe exacerbation of asthma is being managed by you in the emergency department. Which of the following developments would not be consistent with imminent respiratory arrest? a) Absence of wheezes b) Bradycardia c) Pulsus paradoxicus of 15 mm Hg d) Paradoxical thoracoabdominal movement e) Confusion Findings that would suggest imminent respiratory arrest in a patient with a severe asthma exacerbation include the development of drowsiness, paradoxical thoracoabdominal movement, and bradycardia. As a result of respiratory muscle fatigue, loud wheezes may disappear and pulsus paradoxicus may be absent. Pulsus paradoxicus of 10 to 25 mm Hg is consistent with a moderate asthma exacerbation.

An 8-year-old girl presents to the emergency department with severe wheezing over the past 24 hours, and has been treated with 10 puffs of albuterol during that time period. Her vital signs include a respiratory rate of 24 breaths per minute, pulse rate of 112 beats per minute, blood pressure of 124/86 mm Hg, and oxygen saturation (measured by pulse oximetry) of 88%. Her examination reveals dry and slightly cyanotic mucous membranes, prominent use of accessory muscles of respiration, and diffuse bilateral wheezing. Initial steps in her management should include _____________. a) administration of oxygen to maintain oxygen saturation of at least 90% b) insert an IV catheter c) obtain an arterial blood gas analysis d) administer nebulized albuterol 2.5 mg e) All of the above This young girl has acute severe asthma with probable hypoxemia. An arterial blood gas analysis will help establish whether she is hypercapniac, which would alert the physician that her increased respiratory rate and work of breathing have led to fatigue. She requires immediate maintenance of oxygenation and fluid status along with continued treatment of her severe airways obstruction, using nebulized albuterol.

A 25-yr old woman comes to the office complaining of a nonproductive cough that has lasted for 6 months. Findings of the physical exam are unremarkable, and you ask the patient to obtain a chest X-ray, with is normal. When the patient returns with the same symptoms 2 weeds later, which of the following tests should be ordered to rule out hyper-reactive airways (cough-variant asthma)? A. B. C. D. E. Lung ventilation/perfusion scan Methacholine bronchial challenge test Sputum culture and susceptibility studies Lung CT scan Bronchoscopy

Patients with cough-variant asthma may or may not have wheezing, and results of spirometry testing may be normal. A methacholine challenge test that show a significant drop in FEV1 indicates airway hyper-reactivity.

Of these, only _______ has not been found related to increases in asthma prevalence and mortality. A. B. C. D. E. Environmental pollution Adverse effects of asthma medications Limited access to health care Widespread use of antibiotics in infancy Obesity

Studies have linked the severity of asthma symptoms to urban pollution levels, and some evidence suggests that asthma is more common in obese patients. Access to health care clearly affects treatment. Many have argued that the immune hypothesis that is, the overuse of antibiotics in infancy and the resulting lack of interaction between the immune system and foreign antigens -- leads to a hypersensitivity state. While the overuse of isoproterenol HCl to treat asthma was implicated in the rise of asthma deaths in Europe several decades ago, current drug regimens do not appear to increase morbidity or mortality.

A 26-yr old man comes in complaining about his asthma. He says he usually has symptoms 1 day a week and about 2 nights a month and responds well to inhaled albuterol (Proventil). How would you classify his asthma? A. B. C. D. Mild intermittent Moderate intermittent Mild persistent Moderate persistent

Mild intermittent asthma - 2 days a week or less and 2 nights a month or less Mild persistent asthma - less then once a day but more than 2 days a week and more than 2 nights a month Moderate persistent symptoms daily and more than one night a week Severe persistent symptoms are continual during the day and frequent during the night. Moderate intermittent not a classification

Which one of the following medications would bring relief to an asthma patient who has dyspnea? A. B. C. D. E. Inhaled albuterol Inhaled tiotropium bromide (Spiriva) Inhaled fluticasone propionate (Flovent) Intravenous methylprednisolone Inhaled acetylcysteine (Mucomyst)

Albuterol has the most rapid onset of action and is the only reliever medication among those listed. Inhaled tiotropium, fluticasone, and acetylcysteine, as well as intravenous methylprednisolone, are controller medications.

Which one of the following statements is not true? A. Asthma is characterized by recurrent episodes of wheezing, breathlessness, chest tightness and cough. B. Differential diagnosis of asthma includes gastroesophageal reflux disease (GERD), bronchitis and upper airway obstruction due to a foreign body. C. Severe persistent asthma is characterized by frequent nocturnal symptoms. D. Although self-management education can be a useful complement to routine asthma care, there is no evidence indicating that it is more effective than routine care alone. E. Patients who have asthma should be vaccinated against influenza and pneumonia.

Which one of the following statements is not true?


A. Patients who have mild intermittent asthma may need to control their symptoms on a daily basis with a medication such as an inhaled corticosteroid (ICS) or a leukotriene modifier. B. Evidence indicates that inhaled corticosteroids should be used as first-line monotherapy for persistent asthma. C. During follow-up visits with patients who are using beta2agonists, the physician should review dosage instructions and inhaler technique. D. A patient s asthma management plan should include written instructions about when to seek medical care. E. Family physicians should educate patients who have asthma about the importance of avoiding environmental triggers such as tobacco smoke and dust mites.

Which of the following agents is preferred for initiating treatment in children of all ages with persistent asthma? a) Inhaled corticosteroid (ICS) b) Systemic corticosteroid c) Cromolyn d) Leukotriene receptor antagonist (LTRA)

An inhaled corticosteroid (ICS) is the preferred treatment for initiating therapy in children of all ages with severe persistent asthma. Cromolyn or nedocromil are no longer recommended for initial therapy.

Chest Pain

A 23-year-old man who is HIV positive has a 2-week history of midsternal chest pain that is aggravated by eating spicy foods; the pain is unrelated to exertion or position, and he reports no dysphagia. Treatment with H2-receptor blocking agents has provided no relief. He takes clotrimazole lozenges for thrush and zidovudine (AZT). He has a CD4+ T-lymphocyte count of 220/mm3 (Normal 500). Which of the following is the most appropriate next step in management? (A) Therapeutic trial of acyclovir (B) 24-Hour pH probe (C) Acid perfusion test (D) Esophageal manometry (E) Esophagoscopy

A 50-year-old man with hypertension is brought to the emergency department 30 minutes after the sudden onset of severe chest pain that radiates to his back and arms. His blood pressure is 180/80 mm Hg in his left arm; no pressure reading can be obtained from the right arm. Cardiac examination shows a murmur of aortic insufficiency. Which of the following is the most likely diagnosis? (A) Acute aortic dissection (B) Acute myocardial infarction (C) Embolus to the right subclavian artery (D) Pulmonary embolism (E) Spontaneous pneumothorax

A 37-year-old man is brought to the emergency department 1 hour after he acute onset of tearing anterior chest pain. The pain radiates to the left back and arm. He is 203 cm (6 ft 7 in) tall and weighs 86 kg (190 lb); BMI is 21 kg/m2. His temperature is 36.7EC (98EF), pulse is 116/min, respirations are 20/min, and blood pressure is 115/70 mm Hg. The lungs are clear to auscultation. An ECG shows no abnormalities. What is the most likely diagnosis? (A) Angina pectoris (B) Aortic dissection (C) Costochondritis (D) Esophageal spasm (E) Pneumothorax

A 33-year-old woman comes to the emergency department 30 minutes after the sudden onset of chest pain, palpitations, shortness of breath, numbness and tingling in both arms, and fear of going crazy. She has visited local emergency departments several times over the past 3 months for similar symptoms that resolved within 1 hour. She uses an oral contraceptive. She drinks two beers daily and six beers on the weekend. She has no history of medicalproblems. Her mother and sisters have a history of anxiety. Her pulse is 90/min, respirations are 18/min, and blood pressure is 130/90 mm Hg. Physical examination, laboratory studies, and an ECG show no abnormalities. What is the most likely diagnosis? (A) Alcohol withdrawal (B) Hyperthyroidism (C) Major depressive disorder (D) Panic disorder (E) Pulmonary embolus

A previously healthy 18-year-old man comes to the emergency department 12 hours after the onset of chest pain in the area of the trapezius muscle. He has had an upper respiratory tract infection for 9 days. A friction rub is heard over the precordium. An ECG shows an increase in the J point of all leads except aVR and V1. After administration of aspirin, the pain subsides. What is the most likely cause of his symptoms? (A) Acute pericarditis (B) Myocarditis (C) Pleuritis (D) Pulmonary embolism (E) Spontaneous pneumothorax

A 42-year-old woman comes to the emergency department 1 hour after the sudden onset of chest pain, cough, dyspnea, tachypnea, and marked anxiety. Two days ago, she underwent right hemicolectomy for cancer of the ascending colon. An accentuated pulmonary S2 is heard on auscultation. An ECG shows nonspecific ST-segment and T-wave changes. Leukocyte count is 12,000/mm3. An x-ray of the chest shows no pulmonary infiltrates or pleural effusions. Arterial blood gas analysis on room air shows a PCO2 of 30 mm Hg and a PO2 of 55 mm Hg. What is the most likely cause of her symptoms? (A) Acute pericarditis (B) Myocarditis (C) Pleuritis (D) Pulmonary embolism (E) Spontaneous pneumothorax

A 38-year-old Hispanic bank executive comes to the emergency department because of the sudden onset of shortness of breath, light-headedness, diaphoresis, and weakness. He is afebrile. On auscultation of the lungs, bilateral basilar rales are heard. Electrocardiogram is shown. Which of the following is the most likely diagnosis? (A) Acute pericarditis (B) Hyperventilation syndrome (C) Myocardial infarction (D) Pulmonary embolism (E) Spontaneous pneumothorax

Unless contraindicated, all of the following medications except ______ are recommended for patients having unstable angina/non-ST-segments elevation myocardial infarction when they are discharged from the hospital. A) B) C) D) E) Aspirin and/or clopidogrel (Plavix) A B-blocker A statin A angiotensin-converting-enzyme inhibitor A diuretic

Five medications have a New York Heart Association class I recommendation for long-term treatment of patients after unstable angina/non-ST-segment elevation MI. A diuretic does not. Statins should be continued as secondary prevention. Aspirin should be given, and if it is not tolerated, clopidogrel (Plavix) can be used. Clopidogrel can also be given with aspirin for up to 9 months. Bblockers should be continued to control symptoms. ACE inhibitors have shown favorable effects with long-term use in patients with coronary artery disease.

Back Strain/Sprains

A 60-year-old woman comes to the physician because of severe aching and stiffness in her neck, shoulders, and hips for 2 months. Her symptoms are most pronounced in the morning shortly after awakening. She has had chronic fatigue and low-grade fevers during this period. Range of motion of the neck, shoulders, and hips is normal. The muscles are minimally tender to palpation. Muscle strength, sensation, and deep tendon reflexes are normal. Serum creatine kinase activity is 40 U/L, and erythrocyte sedimentation rate is 80 mm/h. Serum rheumatoid factor and antinuclear antibody assays are negative. Which of the following is the most likely diagnosis? (A) Fibromyositis (B) Osteoarthritis (C) Polymyalgia rheumatica (D) Polymyositis (E) Seronegative rheumatoid arthritis

A 35-year-old woman is seen in the emergency department because of the sudden onset of severe low back pain 12 hours earlier. The pain began when she bent over to pick up her 2-year-old child. She has been unable to stay in bed because of the need to care for her child. Low doses of ibuprofen have eased the discomfort slightly. Careful physical examination, including a neurologic examination, is normal except for evidence of muscle spasm. She believes she has a herniated disc because 2 years ago her father developed the sudden onset of back pain that required immediate surgery. Which of the following is the most appropriate first step? (A) Order CT scan of the lumbar spine (B) Order MRI of the lumbar spine (C) Order x-ray films of the lumbar spine (D) Reassure her and treat with a nonsteroidal anti-inflammatory drug (NSAID) (E) Request consultation with an orthopedic surgeon

COPD

A 55-year-old man has dyspnea on exertion over the past 10 months that prevents him from climbing more than one flight of stairs. Examination of the chest shows an increased anteroposterior diameter; there is hyperresonance to percussion. Breath sounds are distant with faint end-expiratory wheezes. Which of the following vaccinations should this patient receive on an annual basis? (A) Clostridium tetani (B) Haemophilus influenzae type b (C) Hepatitis B virus (D) Influenza A virus (E) Streptococcus pneumoniae

A 60-year-old man with emphysema is brought to the emergency department 1 hour after becoming lethargic and incoherent. Over the past 3 days, he has had increasing episodes of dyspnea, cough, and sputum production. Oxygen was administered en route to the hospital. On arrival, the patient is confused and lethargic but responsive to painful stimuli. Arterial blood gas analysis while breathing 6 L/min of oxygen shows: pH 7.20, PCO2 95 mm Hg, PO2 100 mm Hg. Which of the following is the most appropriate next step in management? (A) Discontinuation of the oxygen (B) Continuation of current oxygen with the addition of a respiratory stimulant (C) Administration of 2 L/min of oxygen by nasal prongs (D) Administration of 24% oxygen by Venturi mask (E) Endotracheal intubation and mechanical ventilation

56-year-old woman is undergoing evaluation for newly diagnosed chronic obstructive pulmonary disease. She has smoked one pack of cigarettes daily for 30 years. She has a morning cough productive of phlegm and has had three to four severe upper respiratory tract infections accompanied by shortness of breath every winter. She has a sedentary lifestyle and is 14 kg (30 lb) overweight. Mild expiratory wheezing is heard. There is no cyanosis or pedal edema. Spirometry shows an FEV1 of 62% of predicted, an FVC of 73% of predicted, and an FEV1 to FVC ratio of 65% (N>75%). Which of the following will have the greatest impact on this patient s long-term prognosis? (A) Yearly influenza virus immunization (B) Cessation of smoking (C) Regular exercise program (D) Regular use of a -adrenergic agonist inhaler (E) Daily percussion and postural drainage

Acute Sinusitis

An 18-year-old white high school student comes to the office in late August because of a stuffy nose for 1 week and a toothache and fever. On physical examination he has right facial fullness and pain below his eye when he leans forward. There is tenderness in the region of the upper premolar and molar teeth on the right side. His temperature is 38.7EC (101.6EF), orally. He has bloody, thick, green mucus coming from his right nostril. The remainder of his physical examination is normal. You suspect maxillary sinusitis on the basis of the clinical findings. Before beginning antibiotic treatment in this patient, it is necessary to first do which of the following? (A) Confirm the diagnosis with CT films of the sinus (B) Confirm the diagnosis with plain x-ray films of the sinus (C) Confirm the diagnosis with transillumination of the sinus (D) Request consultation with a dentist (E) No additional steps are necessary

S-ar putea să vă placă și